Nick has recently learned to feed himself. he can scribble and talk in halting words. he continuously wants to experiment with new things and throws temper tantrums at times. which stage of life is nick going through

Answers

Answer 1

Nick is likely going through the toddler stage of development, which typically occurs between the ages of 1-3 years old.

This stage is marked by a desire for independence and exploration, as well as the development of fine motor skills such as feeding oneself and scribbling. Halting words are also typical at this stage as language skills are still developing.

However, it is also common for toddlers to experience temper tantrums as they struggle to communicate their needs and desires. It is important for caregivers to provide a safe and supportive environment for toddlers to explore and learn, while also setting appropriate boundaries and teaching social skills to manage temper tantrums in a positive way.

To know more about stage of development visit:

https://brainly.com/question/29757009

#SPJ11


Related Questions

Ahmad the trainer has two solo workout plans that he offers his clients: Plan A and Plan B. Each client does either one or the other (not both). On Monday there were clients 3 who did Plan A and 2 who did Plan B. On Tuesday there were 5 clients who did Plan A and 6 who did Plan B. Ahmad trained his Monday clients for a total of 3 hours and his Tuesday clients for a total of 7 hours. How long does each of the workout plans last? please help

Answers

Answer:

Plan A last for 30 minutes

Plan B last for 45 minutes

Step-by-step explanation:

There are two plans:

Plan A

Plan B

Monday

Plan A=3 clients

Plan B=2 clients

Total hours=3

Tuesday

Plan A=5 clients

Plan B=6 clients

Total hours=7

The equation for the unknown

3A + 2B=3 (1)

5A + 6B=7 (2)

Multiply (1) by 3 and (2) by (1)

9A + 6B=9 (3)

5A + 6B=7 (4)

Subtract (4) from (3)

4A=2

Divide both sides by 4

4A/4 = 2/4

A=1/2 hours

Substitute A=1/2 into (1)

3A + 2B=3

3(1/2) +2B=3

3/2 + 2B=3

2B= 3 - 3/2

2B= 6-3/2

2B=3/2

Divide both sides by 2

B=3/2÷2

=3/2×1/2

B=3/4 hours

A=1/2 hours

B=3/4 hours

60 minutes=1 hour

A=1/2 hours

1/2 × 60 minutes

=60/2

=30 minutes

B=3/4 hours

=3/4×60 minutes

=180/4

=45 minutes

Plan A last for 30 minutes

Plan B last for 45 minutes

HELP PLEASE MATH ALGEBRA

HELP PLEASE MATH ALGEBRA

Answers

Answer:

1. binomial

2. trinomial

3. binomial

4. monomial

5. monomial

6. trinomial

monomial has only a single value, binomial two and trinomial three values.

a change in a population that is not related strictly to the size of the population is best described as

Answers

A change in a population that is not related strictly to the size of the population can be described as a change in the demographic makeup of the population. This refers to changes in the characteristics of individuals within the population, such as age, gender, education level, and ethnicity.

For example, if a population experiences an influx of young adults, this would represent a change in the demographic makeup of the population, even if the overall size of the population remains the same.

Other factors that can contribute to a change in the population's makeup include migration patterns, changes in birth rates and mortality rates, and shifts in cultural or social norms. These changes can have significant impacts on a population, affecting everything from economic growth to social dynamics. Understanding demographic changes is therefore critical for policymakers and researchers seeking to address issues such as inequality, public health, and urban planning. By analyzing population trends and anticipating future changes, we can better prepare for the challenges and opportunities that lie ahead.

learn more about ethnicity here: brainly.com/question/2906553

#SPJ11

Use π=3.14 and round the answer to the nearest hundredth.

Use =3.14 and round the answer to the nearest hundredth.

Answers

Answer: A = 615.44 m^2

Step-by-step explanation:

area = π · r^2

= 3.14(14)^2

Is 0.3 greater than 0.09

Answers

Answer:

Yes, because 9 is in the hundreths place and 3 is in the tenths

Step-by-step explanation:

Answer:

Step-by-step explanation:

no because 0.3 is the same as 3/10 and 0.9 is the same as 9/10 wich is greater

Given the equation 2

Square root of x minus 5 = 2, solve for x and identify if it is an extraneous solution. (2 points)

Group of answer choices

x = 9, solution is not extraneous

x = 11, solution is extraneous

x = 9, solution is extraneous

x = 11, solution is not extraneous

Answers

Answer:

its none

Step-by-step explanation:

a newspaper boy is trying to perfect his business in order to maximize the money he can save for a new car. daily paper sales are normally distributed, with a mean of 100 and standard deviation of 10. he sells papers for $0.50 and pays $0.30 for them. unsold papers are trashed with no salvage value. how many papers should he order each day? round up.

Answers

To determine how many papers the newspaper boy should order each day, we need to consider his profit margin. His profit is the difference between the revenue earned from selling the papers and the cost of buying them.



The revenue earned is the number of papers sold multiplied by the selling price of $0.50. The cost of buying the papers is the number of papers ordered multiplied by the buying price of $0.30. Let's say he orders x papers each day. The expected value of his revenue can be calculated as x multiplied by the mean of 100 papers,

which is 100x. The expected value of his cost can be calculated as x multiplied by the buying price of $0.30, which is 0.3x. His profit can then be calculated as the difference between his revenue and cost, which is 0.2x (since the selling price of $0.50 minus the buying price of $0.30 is $0.20 profit per paper).



To maximize his profit, he should order the number of papers that gives him the highest expected profit. This occurs at the point where the deviation from the mean is zero. In other words, he should order the number of papers that gives him the highest probability of selling all of them, without having any unsold papers that he needs to throw away.



Using the formula for standard deviation, we can calculate that the probability of selling all 100 papers is 68.3%. The probability of selling 101 papers is slightly lower at 64.2%, while the probability of selling 99 papers is also slightly lower at 64.2%.



Therefore, to maximize his profit, the newspaper boy should order 100 papers each day, since this gives him the highest probability of selling all of them without having any unsold papers. This would give him a daily profit of $10 (100 papers sold x $0.20 profit per paper).

To know more about probability click here

brainly.com/question/15124899

#SPJ11

Find the particular solution to dy dx ex if y(2) = 5. - Select one: 1 a. y = 3 **? + b.y = 3x2 + 4 1 4 c. y = In [x] + 5 - In 2 1 d. y = x 10.5

Answers

The particular solution to the given differential equation with the initial condition y(2) = 5 is y = eˣ + (5 - e²). Therefore, the correct option is c.

To find the particular solution to the given differential equation dy/dx = eˣ with the initial condition y(2) = 5, we can integrate both sides of the equation.

∫dy = ∫eˣ dx

Integrating, we get:

y = eˣ + C

where C is the constant of integration. To find the value of C, we can substitute the initial condition y(2) = 5 into the equation:

5 = e² + C

Solving for C, we have:

C = 5 - e²

Substituting this value of C back into the equation, we obtain the particular solution:

y = eˣ + (5 - e²)

So, the correct option is c.

Learn more about differential equation:

https://brainly.com/question/1164377

#SPJ11

calcaulate the reliability of the system if components a and b function properly with probabilty 0.95; c and d function properly with probability 0.85

Answers

To calculate the reliability of the system, we need to consider the probabilities of each component functioning properly and apply the appropriate reliability model.

In this scenario, components A and B function properly with a probability of 0.95 each, while components C and D function properly with a probability of 0.85 each.

Assuming the components are independent, we can use the multiplication rule to calculate the overall reliability of the system. According to the multiplication rule, the reliability of the system is equal to the product of the reliabilities of its individual components.

Reliability of system = Reliability of A * Reliability of B * Reliability of C * Reliability of D

Reliability of system = 0.95 * 0.95 * 0.85 * 0.85

By multiplying the individual component reliabilities, we can determine the reliability of the entire system. The result will provide a measure of the probability that the system functions properly.

Please note that this calculation assumes the components are independent, meaning the failure or success of one component does not affect the others. Additionally, this calculation assumes a simple serial configuration, where all components must function properly for the system to work. Other system configurations may require different reliability models to be applied.

Learn more about probabilities here:

https://brainly.com/question/32117953

#SPJ11

justin surveyed his freibds about how mant pets they have use the data in the table to find the median mode and range

Answers

Answer:

where is the data from the table?

A polling company conducts an annual poll of adults about political opinions. The survey asked a random sample of 361
adults whether they think things in the country are going in the right direction or in the wrong direction. 47​% said that things were going in the wrong direction.

a) Are the assumptions and conditions required to apply a confidence interval met? Select all that apply.
A. Yes, all assumptions and conditions are met.
B. No, because the sample is a simple random sample.
C. No, because there are less than 10 expected "successes" and 10 expected "failures."
D. No, because the sample is greater than 10% of the population.
E. No, because the sample is less than 10% of the population.
F. No, because there are at least 10 expected "successes" and 10 expected "failures."
G. No, because the sample is not a simple random sample.

Answers

In this case, option (F) is the correct choice, because there are at least 10 expected "successes" and 10 expected "failures" in the sample, which satisfies the requirement for constructing a confidence interval for a proportion.

The question provides information about a survey conducted by a polling company to measure political opinions.

The survey asked a random sample of 361 adults whether they think things in the country are going in the right direction or the wrong direction, and 47% responded that things were going in the wrong direction.

The question is asking whether the assumptions and conditions required to apply a confidence interval are met.

To apply a confidence interval, we assume that the sample is a simple random sample from the population of interest, and that the sample size is sufficiently large.

Moreover, for constructing a confidence interval for a proportion, we also require that there are at least 10 expected "successes" and 10 expected "failures" in the sample.

Option (B) is incorrect because a simple random sample is one of the assumptions required to apply a confidence interval, and the question states that the sample is a random sample.

Option (C) is incorrect because the sample size is large enough for constructing a confidence interval.

Option (D) and option (E) are incorrect because they do not accurately reflect the conditions required to apply a confidence interval for a proportion.

Option (A) and option (G) are not correct choices because they do not accurately address the assumptions and conditions required to apply a confidence interval for a proportion.

Therefore, the correct answer is (F), i.e., the assumptions and conditions required to apply a confidence interval are met, including the requirement of having at least 10 expected "successes" and 10 expected "failures".

For similar question on confidence interval.

https://brainly.com/question/30536583

#SPJ11

2. Determine an equation of the tangent line to the function y=lnx at x=e 2. Sketch a graph of the function and tangent line, marking key points

Answers

Equation of the tangent line: y = x - 2e + 2.The equation of the tangent line to the function y = ln(x) at x = e is y = x - 2e + 2.

To find the equation of the tangent line to the function y = ln(x) at x = e (approximately 2.71828), we need to determine the slope of the tangent line at that point. The slope of the tangent line is equal to the derivative of the function at that point.

Taking the derivative of y = ln(x) with respect to x, we get:

dy/dx = 1/x

Substituting x = e into the derivative, we have:

dy/dx = 1/e

So, the slope of the tangent line at x = e is 1/e.

Next, we use the point-slope form of a linear equation to find the equation of the tangent line. Using the point (e, ln(e)), and the slope 1/e, we get:

y - ln(e) = (1/e)(x - e)

Simplifying this equation gives us the equation of the tangent line:

y = x - 2e + 2

The equation of the tangent line to the function y = ln(x) at x = e is y = x - 2e + 2. The tangent line intersects the curve at x = e and has a slope equal to 1/e. When graphed together, the tangent line will touch the curve at x = e and represent the local linear approximation of the function at that point.

To know more about  tangent line follow the link:

https://brainly.com/question/31306002

#SPJ11

What is the value of x in the equation ? (image below)

What is the value of x in the equation ? (image below)

Answers

Answer: 2

Step-by-step explanation:

2.5(6x-4)=10+4(1.5+0.5x) [Work out the brackets]

15x-10=0+6+2x [Solve]

15x-10=16+2x [Convey the terms]

15x-2x=16+10 [Add up the same terms and calculate]

13x=26 [Divide both parts]

x=2

Answer:

x = 2

General Formulas and Concepts:

Pre-Algebra

Order of Operations: BPEMDASEquality Properties

Step-by-step explanation:

Step 1: Define equation

2.5(6x - 4) = 10 + 4(1.5 + 0.5x)

Step 2: Solve for x

Distribute:                                   15x - 10 = 10 + 6 + 2xCombine like terms:                   15x - 10 = 16 + 2xSubtract 2x on both sides:        13x - 10 = 16Add 10 to both sides:                 13x = 26Divide both sides by 13:             x = 2

Step 3: Check

Plug in x into the original equation to verify it's a solution.

Substitute in x:                    2.5(6(2) - 4) = 10 + 4(1.5 + 0.5(2))Multiply:                               2.5(12 - 4) = 10 + 4(1.5 + 1)Subtract/Add:                      2.5(8) = 10 + 4(2.5)Multiply:                               20 = 10 + 10Add:                                     20 = 20

Here we see that 20 does indeed equal 20.

∴ x = 2 is a solution of the equation.

THIS 50 POINTS ANSWER FAST. Graph two lines whose solution is (1,4) give me the 2 of the equations. For the graphs go to Desmos. Answer fast.

Answers

Answer:

The graph is shown below.

Step-by-step explanation:

THIS 50 POINTS ANSWER FAST. Graph two lines whose solution is (1,4) give me the 2 of the equations. For

The growth of the world's population can be represented as A = Aoert, where A is the population at time t, Ao is the
population at t = 0, and r is the annual growth rate. The world's population at the beginning of 2008 was estimated at 6,641,000,000. If
the annual growth rate is 1.2%, in what year will the world population reach 9 billion?

The growth of the world's population can be represented as A = Aoert, where A is the population at time

Answers

We can use the given population growth formula:

A = Ao * e^(r*t)

Let's first find out what the value of t is when the population reaches 9 billion:

9,000,000,000 = 6,641,000,000 * e^(0.012*t)

Dividing both sides by 6,641,000,000, we get:

1.355 = e^(0.012*t)

Taking the natural logarithm of both sides, we get:

ln(1.355) = 0.012*t

Solving for t, we get:

t = ln(1.355) / 0.012 ≈ 41.8

the world population will reach 9 billion in the year 2050 (2008 + 41.8).

(If this doesn’t seem right to you make sure you comment!)

how will the interval compare to the interval calculated for the same percentage if a normal distribution is assumed and the Empirical Rule is used?

Answers

The 68-95-99.7 rule, also known as the empirical rule, is a shorthand for remembering the percentage of values in a normal distribution that lie within an interval estimate: 68%, 95%, and 99.7% of values lie within one, two, and three standard deviations of the mean, respectively.

What is empirical rule?

The empirical rule, also known as the three-sigma rule or the 68-95-99.7 rule, is a statistical rule stating that for a normal distribution, nearly all observed data will fall within three standard deviations (denoted by σ) of the mean or average (denoted by µ).

The Empirical Rule states that 99.7% of data from a normal distribution is within 3 standard deviations of the mean.

68% of the data is within one standard deviation of the mean, 95% is within two standard deviations, and 99.7% is within three standard deviations, according to this rule.

Three-sigma limits that follow the empirical rule are used to define the upper and lower control limits in statistical quality control charts and risk analysis such as VaR.

Learn more about standard deviation

https://brainly.com/question/23907081

#SPJ1

Write the equation of the line in standard form.
y = 3x + 10
X
x
X
X
A
B
C
D
3x + y = -10
3x + y = 10
3x - y = -10
3x - y = 10

Answers

The equation of the line y = 3x + 10 in standard form is 3x - y = -10.

What is the equation of line in standard form?

Given the equation of line in the question:

y = 3x + 10

To write the equation of the line in standard form, we need to rearrange the equation so that the x and y terms are on the left side and the constant term is on the right side of the equation.

It is expressed as;

Ax + Bx = C

y = 3x + 10

Subtracting 3x from both sides, we get:

y - 3x = 3x - 3x + 10

y - 3x = 10

Reorder

-3x + y = 10

Multiply each term by -1

-3x(-1) + y(-1) = 10(-1)

3x - y = -10

Therefore, the equation in standard form is 3x - y = -10.

Learn more about equation of line here:https://brainly.com/question/28696396

#SPJ1

A rancher wishes to fence in a rectangular corral enclosing 1300 square yards and must divide it in half with a fence down the middle. If the perimeter fence costs $5 per yard and the fence down the middle costs $3 per yard, determine the dimensions of the corral so that the fencing cost will be as small as possible.'

Answers

The dimensions of the corral that will minimize the cost of the fencing are x = 4 yards (width) and y = 325 yards (length).

To begin solving this problem, we need to use the given information to set up an equation that represents the cost of the fencing. Let's start by defining the dimensions of the rectangular corral. We can use x to represent the width and y to represent the length.

Since the area of the corral is 1300 square yards, we know that:

xy = 1300

Now, let's think about the fencing. We need to divide the corral in half with a fence down the middle, which means we have two equal sections with a width of x/2. The length of each section is still y.

To find the perimeter of each section, we add up all the sides. For the top and bottom, we have two lengths of y and two widths of x/2. For the sides, we have two lengths of x/2 and two widths of y. This gives us a perimeter of:

2y + x + 2x + 2y = 4y + 2x

Since we have two sections, the total perimeter is:

2(4y + 2x) = 8y + 4x

We can now set up an equation for the cost of the fencing:

Cost = (8y + 4x)($5) + (x)($3)

The first part of the equation represents the cost of the perimeter fence, while the second part represents the cost of the fence down the middle.

Now, we want to find the dimensions of the corral that will minimize the cost of the fencing. To do this, we can use calculus. We take the derivative of the cost equation with respect to x and set it equal to zero:

dCost/dx = 20y + 3 = 0

Solving for y, we get:

y = -3/20

Since we can't have a negative length, this solution is not valid. However, we can find the minimum cost by plugging in the value of y that makes the derivative equal to zero into the original equation for the cost of the fencing. This gives us:

Cost = (8y + 4x)($5) + (x)($3)
Cost = (8(-3/20) + 4x)($5) + (x)($3)
Cost = (-(12/5) + 4x)($5) + (x)($3)
Cost = -24x + 3x^2 + 3900

To minimize the cost, we take the derivative with respect to x and set it equal to zero:

dCost/dx = -24 + 6x = 0
x = 4

Plugging this value of x back into the equation for the cost of the fencing gives us:

Cost = -24(4) + 3(4^2) + 3900
Cost = $3892

Therefore, the dimensions of the corral that will minimize the cost of the fencing are x = 4 yards (width) and y = 325 yards (length).

To learn more about dimensions, refer here:

https://brainly.com/question/28688567#

#SPJ11

PLEASE HELP ME (third times the charm)!!!!!!!

PLEASE HELP ME (third times the charm)!!!!!!!

Answers

Answer:

the first option i think

Which equation matches the table?

Which equation matches the table?

Answers

An equation that matches the table include the following: y = x + 5.

How to determine an equation of this line?

In Mathematics and Geometry, the point-slope form of a straight line can be calculated by using the following mathematical equation (formula):

y - y₁ = m(x - x₁)

Where:

m represent the slope.x and y represent the points.

First of all, we would determine the slope of this line;

Slope (m) = (y₂ - y₁)/(x₂ - x₁)

Slope (m) = (6 - 5)/(1 - 0)

Slope (m) = 1/1

Slope (m) = 1.

At data point (0, 5) and a slope of 1/, a linear equation in slope-intercept form for this line can be calculated by using the point-slope form as follows:

y - y₁ = m(x - x₁)

y - 5 = 1(x - 0)

y - 5 = x

y = x + 5

Read more on point-slope here: brainly.com/question/24907633

#SPJ1

3(x+6)+2x-5=-2(x+1)+10

Answers

Step-by-step explanation:

3x+18+2x-5= -2x+-2+10

combining like terms

3x+2x+2x=5-2+10-18

7x=-5

x=-5/7

3x+18+2x-5=-2x-2+10
5x+13=-2x-2+10
5x+13=-2x+8
5x+13+2x=8
7x+13=8
7x=8-13
7x=-5
the answer is
x=-5/7

find the cost of 10 pens at x pence each

Answers

Answer:

D. x + 10 pence

Step-by-step explanation:

Since the pen costs 10 pence more than the ruler, we need to add 10 to x.

2a) Determine the unknown angle x.

2a) Determine the unknown angle x.

Answers

The unknown angle x in the triangle is 80 degrees

How to determine the unknown angle x.

From the question, we have the following parameters that can be used in our computation:

The triangle

The unknown angle x is calculated using the sum of angles in a triangle theorem

So, we have

x + 60 + 40 = 180

Evaluate the like terms

x + 100 = 180

So, we have

x = 80

Hence, the unknown angle x is 80 degrees

Read more about angles at

https://brainly.com/question/31898235

#SPJ1

guys can you help me with the like terms plz plz dont answer if you dont know it

guys can you help me with the like terms plz plz dont answer if you dont know it

Answers

Answer:

like terms are numbers or letters that are a like.

y² is the only letter with a square in this problem. Whilst xy and 2xy are the same.

y²+ xy + 2xy

y² there is no letter with square again.

xy+2xy is the same as one orange + two orange.

1 is never written against a letter.

y²+ xy + 2xy

y²+3xy

Select the simplification that accurately explains the following statement. 7 3 = 7 1 3 A.

Answers

The simplification that accurately explains the statement 73 = 713A is A = 73/713

Selecting the simplification that accurately explains the statement.

From the question, we have the following parameters that can be used in our computation:

7 3 = 7 1 3 A.

Express properly

So, we have

73 = 713A

Divide both sides of the equation by 713

So, we have

A = 73/713

Hence, the simplification that accurately explains the statement is A = 73/713

Read more about expression at

https://brainly.com/question/15775046

#SPJ1

Pls help 15. Find the exact area of the shaded region of the circle shown in the diagram.

Pls help 15. Find the exact area of the shaded region of the circle shown in the diagram.

Answers

Answer:

The shaded region is 9.83 cm²

Step-by-step explanation:

Refer to attached diagram with added details.

Given

Circle O with:

OA = OB = OD - radiusOC = OD = 2 cm

To find

The area of segment ADB.

Solution

Since r = OC + CD, the radius is 4 cm.

Consider right triangles OAC or OBC:

They have one leg of 2 cm and hypotenuse of 4 cm, so the hypotenuse is twice the short leg.

Recall the property of 30°x60°x90° triangle:

a : b : c = 1 : √3 : 2, where a- short leg, b- long leg, c- hypotenuse.

It means OC: OA = 1 : 2, so angles AOC and BOC are both 60° as adjacent to short legs.

In order to find the shaded area we need to find the area of sector OADB and subtract the area of triangle OAB.

Area of sector:

A = π(θ/360)r², where θ- central angle,A = π*((mAOC + mBOC)/360)*r²,A = π*((60 + 60)/360))(4²) =  16.76 cm².

Area of triangle AOB:

A = (1/2)*OC*(AC + BC),  AC = BC = OC√3 according to the property of 30x60x90 triangle.A = (1/2)(2*2√3)*2 = 4√3 = 6.93 cm²

The shaded area is:

A = 16.76 - 6.93 = 9.83 cm²
Pls help 15. Find the exact area of the shaded region of the circle shown in the diagram.

Graph the system of equations. y = 2x y = –x + 6 Two lines on a coordinate plane that intersect at the point 2 comma 4. One line has y intercept 0 and the other has y intercept 6. Two lines on a coordinate plane that intersect at the point negative 2 comma negative 4. One line has y intercept 0 and the other has y intercept negative 6. Two lines on a coordinate plane that intersect at the point 1 comma 2. One line has y intercept 0 and the other has y intercept 3. Two lines on a coordinate plane that intersect at the point 3 comma 3. One line has y intercept 0 and the other has y intercept 6.

Answers

The solution to the systems of equations graphically is (2, 4)

Solving the systems of equations graphically

From the question, we have the following parameters that can be used in our computation:

y = 2x

y = -x + 6

Next, we plot the graph of the system of the equations

See attachment for the graph

From the graph, we have solution to the system to be the point of intersection of the lines

This points are located at (2, 4)

Read more about equations at

brainly.com/question/148035

#SPJ1

Graph the system of equations. y = 2x y = x + 6 Two lines on a coordinate plane that intersect at the

Equation with angle addition can someone please answer I need help please

Equation with angle addition can someone please answer I need help please

Answers

Answer:

99°

Step-by-step explanation:

We know that angles on a straight line add up to 180° so we can set up an equation to find the value of x.

In an equation it's important to do the operation you want to on both sides because if you do an operation on one side only you will end up changing the whole equation and that will make the value of x wrong so its an important rule to remember.

8x - 13 + 7x - 17 = 180

→ Simplify

15x - 30 = 180

→ Add 30 to both sides to isolate 15x

15x = 210

→ Divide both sides by 15 to isolate x

x = 14

But we are not finished, the specifies us to find ∠MON and we have only found the value of x so we have to substitute x back into 8x - 13 to find the value of ∠MON

8x - 13

→ Substitute x = 14 back into into the expression

8 × 14 - 13

→ Simplify

112 - 13 = 99

So the value of ∠MON is 99°

HELP ME WITH MY MATH

HELP ME WITH MY MATH

Answers

Answer:

91 hundreths, (1*1)+(9*1/10), .19

<3 love from maddie

Answer:

(1*1)+(9*1/10) then 91 hundreths and last 0.19

Hope it helps!

A sphere has a volume that is 36 cubic meters. Find the radius of the sphere.

A sphere has a volume that is 36 cubic meters. Find the radius of the sphere.

Answers

Answer:

2.05m

Step-by-step explanation:

Other Questions
the lifetime of a printer costing 250 is exponentially distributed with mean 2 years. the manufacturer agrees to pay a full refund to a buyer if the printer fails during the first year following its purchase, and one-half refund if it fails during the second year. if the manufacturer sells 100 printers, how much should it expect to pay in refunds? which type of figurative language is in the phrase, "in over his head"? g why are we able to study this particular series in lab, but not the other two series? why are we able to study this particular series in lab, but not the other two series? there was no particular reason this series was chosen, but the experiment would be too long if all three series were included. the series we are studying is in the visible light spectrum, while the other series are not, and would thus require special equipment under normal circumstances (such as those in this week's lab), the only light a hydrogen atom will emit comes from the chosen series 1.245 g Ni , 5.381 gI A chemist decomposes samples of several compounds; the masses of their constituent elements are listed. Calculate the empirical formula for each compound. Rhetorical questions used in the old major speech the supreme court determined that enslaved people were property and therefore had no legal rights in . High-quality energy can best be characterized as ____.a. electromagneticb. concentratedc. fossilizedd. kinetice. pure Where is the armory museum located?- in the kremlin- in red square- in the palace of the tsar- in st. petersburgthe answer is kremlin How is the new type of solar panel described in the section titled The spark more useful for the Huichol people than previous versions of solar energy technology? Use two details from the article to support your response. * The _____________ approach to criminology is based on the assumption that human behavior is a product of biological, economic, psychological, and social factors, and that the scientific method can be applied to ascertain the causes of individual behavior. HELP ASAP PLEASE!!!! 1. Create a unique equation using the graphic above with the solution (answer) of 36. There are many possible equations you could create to equal 36 using the chart above. 2. Describe the process you used to create your equation in prompt 1. You do a search on your mobile phone for ""clothing stores near me"". your top result is a search ad for a banana republic (clothing store) that is two blocks away. your friend (standing next to you) does the same search at the same time but does not receive that ad. you are a frequent banana republic shopper and your friend is not. this ad targeting is an example of location-based targeting combined with 26. the nurse is evaluating the effectiveness of the otic medication used for oe. what will the nurse evaluate to determine if the goal of therapy has been met? -13.5=2.7s (im class so hurry) Is Japanese Feudalism an example of militarism? Explain.Was The Opium War an example of militarism? Explain.Was The Annexation of Hawaiian an example of militarism? Explain. student must choose one course for each of periods 1,2, and 3 (they have a spare period 4). Based on the course selections listed, how many different schedules can be made if: 1. There are no restrictions on what courses they can take? Buffer capacity is highest when there is more acid than base. when both the acid and base are strong. when a strong acid and weak base are used. there is no acid or base. there are equal amounts of acid and base. there is more base than acid. Part B Buffer resist changes in pH when all of these answers are correct the buffer solution is diluted base is added to it acid is added to it Part C A buffer containing 0.2 M acetic acid (Ka = 1.8 105) and 0.2 M sodium acetate has a pH of 7.00 5.06 4.74 Cannot determine the pH from the data given 4.46 None of these answers are correct Part D A buffer containing a higher concentration of sodium acetate than acetic acid would have a pH that is equal to the pKa lower than the pKa more information is needed to determine this higher than the pKa Part E A buffer could be made from Check all that apply. a weak acid and a strong base a weak base and a strong acid a weak base and its conjugate acid a strong base and a strong acid none of these answers are correct a weak base and a weak acid (unconjugated) all of these combinations can be a buffer solution Which statement about the theory of Social Darwinism is FALSE?a. The theory argued that failure to advance in society indicated a lack of character.b. The theory borrowed language from Charles Darwin.c. The theory argued that evolution was as natural a process in human society as in nature and that government must not interfere.d. The theory argued that freedom required frank acceptance of inequality.e. The theory argued that the "deserving poor" only included children. If ronnie paid for $40 for 8 hamburgers, find the unit price of a book La cantidad de camisas que poseo hoy es ms del doble delos que posea hace 5 aos. Exprese la relacin entre ambascantidades en funcin del nmero actual de libros